1

色数からの挑戦状にやなさんのかわりにずんだもんがチャレンジしてみた

234
0
$$$$

問題について

参照↓
https://x.com/atdy6gzgzoywrwz/status/1826273690511925428?s=46

\begin{equation} {}_{2}F_{1}\left(a,b;c|z\right)=\left(1-z\right)^{-a}-az\sum_{n=0}^{\infty}\frac{\left(a+1\right)_{n}}{n!\left(c+n\right)}z^{n}-\frac{az}{c}\sum_{k=1}^{c-b-1}{}_{2}F_{1}\left(a+1,b+k;c+1|z\right) \end{equation}

(1)
\begin{eqnarray} \left\{ \begin{array}{l} \left(1-z\right)^{-a}=1+\sum_{n=1}^{\infty}\frac{\left(a\right)_{n}}{n!}z^{n}\\ az\sum_{n=0}^{\infty}\frac{\left(a+1\right)_{n}}{n!\left(c+n\right)}z^{n}=\sum_{n=1}^{\infty}\frac{\left(a\right)_{n}}{\left(n-1\right)!\left(c+n-1\right)}z^{n}\\ \frac{az}{c}\sum_{k=1}^{c-b-1}{}_{2}F_{1}\left(a+1,b+k|z\right)=\sum_{n=1}^{\infty}\frac{\left(a\right)_{n}}{\left(c\right)_{n}}\frac{z^{n}}{\left(n-1\right)!}\sum_{k=1}^{c-b-1}\left(b+k\right)_{n-1} \end{array} \right. \end{eqnarray}
を用いるのだ。
\begin{eqnarray} \left(1-z\right)^{-a}-az\sum_{n=0}^{\infty}\frac{\left(a+1\right)_{n}}{n!\left(c+n\right)}z^{n}=1+\sum_{n=1}^{\infty}\frac{\left(a\right)_{n}}{n!}\frac{c-1}{\left(c+n-1\right)}z^{n} \end{eqnarray}
\begin{eqnarray} {}_{2}F_{1}\left(a,b;c|z\right)+\frac{az}{c}\sum_{k=1}^{c-b-1}{}_{2}F_{1}\left(a+1,b+k|z\right)=1+\frac{\left(a\right)_{n}}{n!}\frac{1}{\left(c\right)_{n}}\{\left(b\right)_{n}+n\sum_{k=1}^{c-b-1}\left(b+k\right)_{n-1}\} \end{eqnarray}
$z^{n}$の係数比較を行うことで下記の式を得るのだ。
\begin{eqnarray} \left(c-1\right)_{n}=\{\left(b\right)_{n}+n\sum_{k=1}^{c-b-1}\left(b+k\right)_{n-1}\} \end{eqnarray}
以下この式を証明するのだ。
ただし、面倒なので$c\rightarrow c+1$としておく。つまり、次の式を証明する。
\begin{eqnarray} \left(c\right)_{n}=\{\left(b\right)_{n}+n\sum_{k=1}^{c-b}\left(b+k\right)_{n-1}\} \end{eqnarray}
【n=1】
\begin{eqnarray} \left(左辺\right)&=&c\\ \left(右辺\right)&=&b+c-b=c \end{eqnarray}
【1,2,...,nまで正しいと仮定】
\begin{eqnarray} \left(c\right)_{n+1}&=&c\{\left(b+1\right)_{n}+n\sum_{k=1}^{c-b}\left(b+k+1\right)_{n-1}\}\\ &=&\left(b\right)_{n+1}+\left(c-b\right)\left(b+1\right)_{n}+nc\sum_{k=1}^{c-b}\left(b+k+1\right)_{n-1} \end{eqnarray}
つまり、次の式を示せばよいのだ。
\begin{eqnarray} \left(c-b\right)\left(b+1\right)_{n}+nc\sum_{k=1}^{c-b}\left(b+k+1\right)_{n-1}=\left(n+1\right)\sum_{k=1}^{c-b}\left(b+k\right)_{n} \end{eqnarray}
これを示すためにさらに、$c-b=M$と置くのだ。
すると示すべき式は次の様に書き直せるのだ。
\begin{eqnarray} M\left(b+1\right)_{n}+n\left(b+M\right)\sum_{k=1}^{M}\left(b+k+1\right)_{n-1}=\left(n+1\right)\sum_{k=1}^{M}\left(b+k\right)_{n} \end{eqnarray}
【M=1の場合】
\begin{eqnarray} \left(左辺\right)&=&\left(b+1\right)_{n}+n\left(b+1\right)\left(b+2\right)_{n-1}\\ &=&\left(b+1\right)_{n}+n\left(b+1\right)_{n}\\ &=&\left(n+1\right)\left(b+1\right)_{n}=\left(右辺\right) \end{eqnarray}
【1,2,...,Mまで成り立つとした場合】
\begin{eqnarray} \left(M+1\right)\left(b+1\right)_{n}+n\left(b+M+1\right)\sum_{k=1}^{M+1}\left(b+k+1\right)_{n-1}&=&M\left(b+1\right)_{n}+n\left(b+M\right)\sum_{k=1}^{M}\left(b+k+1\right)_{n-1}\\ &+& \left(b+1\right)_{n}+n\left(b+M+1\right)\left(b+M+2\right)_{n-1}+n\sum_{k=1}^{M}\left(b+k+1\right)_{n-1}\\ \end{eqnarray}
改行後の式に着目するのだ。
\begin{eqnarray} \left(b+1\right)_{n}+n\left(b+M+1\right)\left(b+M+2\right)_{n-1}+n\sum_{k=1}^{M}\left(b+k+1\right)_{n-1}&=& \left(b+1\right)_{n}+n\left(b+M+1\right)_{n}+n\sum_{k=1}^{M}\left(b+k+1\right)_{n-1}\\ \end{eqnarray}
さらに次の事を示すのだ。
\begin{equation} \left(b+1\right)_{n}+n\left(b+M+1\right)_{n}+n\sum_{k=1}^{M}\left(b+k+1\right)_{n-1}=\left(n+1\right)\left(b+M+1\right)_{n} \end{equation}
【M=0の場合】
\begin{equation} \left(b+1\right)_{n}+n\left(b+1\right)_{n}=\left(n+1\right)\left(b+1\right)_{n} \end{equation}
【M=1の場合】
\begin{eqnarray} \left(b+1\right)_{n}+n\left(b+2\right)_{n}+n\sum_{k=1}^{1}\left(b+k+1\right)_{n-1}&=& \left(b+2\right)_{n-1}\{b+1+n\left(b+n+1\right)+n\}\\ &=& \left(b+2\right)_{n-1}\left(n+1\right)\left(n+b+1\right)\\ &=& \left(n+1\right)\left(b+2\right)_{n} \end{eqnarray}
【0,1,2,...,M-1まで成り立つと仮定】
\begin{eqnarray} \left(b+1\right)_{n}+n\left(b+M+1\right)_{n}+n\sum_{k=1}^{M}\left(b+k+1\right)_{n-1}&=& \left(b+1\right)_{n}+n\sum_{k=1}^{M-1}\left(b+k+1\right)_{n-1}+n\left(b+M+1\right)_{n}+n\left(b+M+1\right)_{n-1}\\ &=& \left(n+1\right)\left(b+M\right)_{n}-n\left(b+M\right)_{n}+n\left(b+M+1\right)_{n}+n\left(b+M+1\right)_{n-1}\\ &=& \left(b+M+1\right)_{n-1}\{b+M+n\left(b+n+M\right)+n\}\\ &=&\left(n+1\right)\left(b+M+1\right)_{n} \end{eqnarray}
このことから、色数君の式は証明されたのだ!

投稿日:822

この記事を高評価した人

高評価したユーザはいません

この記事に送られたバッジ

バッジはありません。

投稿者

コメント

他の人のコメント

コメントはありません。
読み込み中...
読み込み中